Skip to Main Content

PrepTest 79, Game 2, Question 9

Transcript

Which one of the following rangers cannot be assigned to area 3? So you want to figure out under any circumstances what cannot be assigned to area 3. So the correct answer is D, K. K cannot be assigned to area 3 so if we have M in area 3, cuz one of our rules automatically places M in area 3.

And then if we place K in area 3, we're gonna run into a problem meeting the requirements of this rule. So L must be paired with M or K. So it cannot be compared with both or paired with both. So if we put K and M together it's impossible to meet the requirements of this rule, so K and M cannot go together.

So that wasn't an original inference made in this sort of creation of the master diagram. But that's something that you can add to your diagram when you encounter a question like this so this tells us that definitively M and K can never go together, you can go ahead and as from a not law here that says K can never go in area three, so D is our correct answer.

Read full transcript